Sei sulla pagina 1di 67

Practice Questions 1

C1 C3 1
1. A patient has developed congestive heart failure after experiencing his first
myocardial infarction. The pulmonary signs and symptoms you expect to find
include:
A. cough productive of thick yellow secretions.
B. crackles and clubbing of the digits.
C. crackles and cough.
D. inspiratory wheezing and shortness of breath.

C2 C8 1
2. A newborn is examined at birth using the APGAR test. The APGAR does NOT
include:
A. heart rate, blood pressure, respiratory rate.
B. heart rate, respiratory rate, color.
C. muscle tone, heart rate, reflex irritability.
D. reflexes, muscle tone, and color.

C3 C7 3
3. A physical therapist requested that a PTA perform ultrasound on the left shoulder
of a patient. During the treatment session, the patient experienced an electrical
shock. The physical therapist would not be responsible for any injury to the
patient if this was the result of:
A. failure of the PTA to use a ground fault interrupter.
B. failure of the PTA to use sufficient ultrasound gel.
C. product liability.
D. the patient touching the ultrasound machine.

C4 C6 3
4. A 55 year-old individual with documented coronary artery disease but no history
of myocardial infarction is enrolled in an exercise class that utilizes circuit-
interval training. The MOST appropriate spacing of work-rest intervals to stress
the aerobic system is:
A. 1:11/2
B. 1:6
C. 1:7
D. 1:10

C5 C1 3
5. The BEST INITIAL intervention to improve functional mobility in an individual
with a stable humeral neck fracture is:
A. active resistive ROM.
B. isometrics for all shoulder musculature.
C. modalities to decrease pain.

474800226.doc
2 Practice Questions

D. pendulum exercises.

C6 C1 3
6. A twenty-one year-old female dancer with unilateral spondylolysis at L4 is
referred to you. She complains of generalized low back pain when she stands
longer than one hour. A postural screen shows excessive lumbar lordosis, right
hip abduction and internal/medial rotation. Interventions for the subacute phase
should include:
A. stretching the abdominals and hip abductors; strengthening the multifidi.
B. stretching the gluteus medius and maximus; strengthening the abdominals.
C. stretching the iliopsoas and iliotibial (IT) band; strengthening the abdominals.
D. stretching the multifidi and right hip abductors; strengthening the hip
abductors.

C7 C1 1
7. A patient has fixed forefoot varus malalignment. Possible compensatory
motion(s) or posture(s) might include:
A. excessive midtarsal or subtalar pronation.
B. genu recurvatum.
C. hallux varus.
D. ipsilateral pelvic external rotation.

474800226.doc
Practice Questions 3

C8 C7 3
8. The purpose of ABC Physical Therapy Center is to deliver excellent health care
services in a unique, individualized and participative manner at a reasonable cost.
The provision of services under these conditions requires and encourages creative
involvement of physicians, staff, patients and their agents in all aspects of the care
planning and delivery process. This is an example of a:
A. excessive midtarsal or subtalar pronation.
B. strategic plan.
C. values statement.
D. vision statement.

C9 C5 3
9. A 26 year-old presents with weakness of the knee resulting from an anterior
cruciate ligament injury. Your examination reveals moderate pain (5/10) and
excessive translation of the tibia during active knee extension. You determine
functional electrical stimulation (FES) is an appropriate modality. Your protocol
for strengthening the quadriceps and improving stability of the knee should
consist of stimulation of the:
A. hamstrings immediately before the quadriceps to produce cocontraction.
B. hamstrings only.
C. quadriceps immediately before the hamstrings to produce cocontraction.
D. quadriceps only.

C10 C2 1
10. You are reviewing a hospital record prior to examining a patient for the first time.
The suspected diagnosis is multiple sclerosis. On the neurologist’s note you find
the following: DTR right quadriceps is 2+, left quadriceps is 3+. You will focus
your examination of this patient based upon the determination that:
A. both sides are abnormal and indicative of hyporeflexia.
B. both sides are abnormal and indicative of upper motor neuron syndrome.
C. both sides are within normal range and not indicative of pathology.
D. there is a normal response on the right while the left is exaggerated and
indicates spastic hypertonia.

C11 C3 1
11. A 72 year-old patient has an episode of syncope in the physical therapy clinic.
You attempt to rule out orthostatic hypotension as the cause of her fainting. This
is BEST done by:
A. checking HR and BP at rest, and after 3 and 5 minutes of cycle ergometry
exercise.
B. checking HR and BP in supine after 5 minutes rest, then repeating in semi-
Fowler position.

474800226.doc
4 Practice Questions

C. checking resting BP and HR in sitting, then repeating measurements after


standing for 1 minute.
D. palpating the carotid arteries and taking HR; using the supine position for BP
measurements.

C12 C4 3
12. A patient with a grade III diabetic foot ulcer is referred for physical therapy. The
wound is foul smelling with a green exudate. You describe this type of drainage
in the medical record as:
A. purulent.
B. sanguineous.
C. serosanguineous.
D. serous.

C13 C8 1
13. You receive a referral for evaluation of fall risk in a 82 year-old who lives alone
and has had two recent falls. The activity that represents the MOST common risk
factor associated with falls in the elderly is:
A. climbing on a stepstool to reach overhead objects.
B. dressing while sitting on the edge of the bed.
C. turning and sitting down.
D. walking with a roller walker with hand brakes.

474800226.doc
Practice Questions 5

C14 C6 1
14. You are performing a prosthetic checkout on a patient with a transfemoral
amputation. The prosthesis has been fitted with a quadrilateral socket. A checkout
of the walls of the socket should reveal:
A. anterior and lateral walls are 21Ú2 to 3 inches higher than the posterior and
medial walls to ensure proper positioning on the ischial seat.
B. the height of the posterior wall is 2 inches less than all the other walls.
C. the medial wall is 21/2 inches higher than the posterior wall while the anterior
and lateral walls are the same height.
D. the posterior and lateral walls are 2 inches higher than the medial and anterior
walls.

C15 C7 3
15. A two month-old child with bilateral hip dislocations is being discharged home
from your acute pediatric facility. You have developed a home exercise program
and now need to instruct her parents. The MOST important item to assess before
instructing the parents is:
A. the financial reimbursement plan.
B. the home environment.
C. their degree of anxiety and attention.
D. their level of formal education.

C16 C7 3
16. The grip strength of a group of 50 to 60 year-olds was investigated. A mean score
of 40, SD of 5, and range of 26-57 were reported. The grip strength score for a
given patient was determined to be 34. You can safely conclude that in a normal
distribution this patient’s score fell within:
A. 32%.
B. 68%.
C. 75%.
D. 95%.

C17 C3 1
17. Which of the following findings is NOT typically present with cystic fibrosis?
A. excessive weight gain due to decreased ability to exercise.
B. frequent respiratory infections caused by Staph and Pseudomonas.
C. increased pancreatic secretions which cause gastrointestinal obstruction.
D. increased pulmonary secretions which cause airway obstruction.

C18 C8 1
18. An infant demonstrates that the ATNR is NOT obligatory when he/she can:
A. turn the head and bring the hand to mouth on the same side.
B. turn the head to either side.

474800226.doc
6 Practice Questions

C. turn the head to one side and bring the opposite hand to mouth.
D. turn the head to one side and look at the extended arm on that side.

C19 C4 3
19. A 75 year-old patient is referred to physical therapy for back pain. Medical tests
reveal he is at the end stage of pancreatic cancer. The physician has told him he
has cancer but has chosen not to inform the patient about the prognosis. If the
patient asks you what his prognosis is, your BEST response would be to:
A. ethically you must tell him the prognosis.
B. tell him everything should work out for the best.
C. tell him to ask the nurse.
D. tell him to discuss his concerns with the physician.

C20 C1 1
20. During surgery to remove an apical lung tumor, the long thoracic nerve was
injured. The muscle that is weakened (3+/5) and the best position to strengthen it
is:
A. serratus anterior, standing, performing wall push-ups.
B. serratus anterior, supine using weights.
C. subscapularis, supine using a pulley.
D. upper trapezius, standing using hand weights.

C21 C1 2
21. A physical therapist examination reveals: iliac crests high on the left; PSIS low
and posterior on the left; ASIS high and anterior on the left; standing flexion test
shows that the left PSIS moves first and farthest superiorly; Gillet’s test
demonstrates the left PSIS moves inferiorly and laterally less than right; long
sitting test shows the left malleolus moves short to long; sitting flexion test is
negative. In light of the above findings, your diagnosis is:
A. iliac inflare on the left.
B. left anterior rotated innominate.
C. left posterior rotated innominate.
D. left upslip.

C22 C7 3
22. The highest level of provider risk in reimbursement is related to:
A. the capitation payment method.
B. the cost-based payment method.
C. the fee-for-service payment method.
D. the per diem payment method.

C23 C5 3

474800226.doc
Practice Questions 7

23. A patient presents with partial and full thickness burns on the chest and neck
region. You decide to apply TENS prior to your debriding procedure to modulate
pain. The type of TENS for optimal relief is:
A. acupuncture-like (low rate) TENS.
B. brief intense TENS.
C. conventional (high rate) TENS.
D. modulated TENS.

C24 C2 3
24. A patient recovering from traumatic brain injury demonstrates instability during
feeding while sitting in a wheelchair. You determine modification is necessary to
ensure function. The FIRST body segment or segments you would align is the:
A. head.
B. lower extremities.
C. pelvis.
D. trunk.

C25 C2 2
25. Your patient demonstrates beginning recovery Stage 4 movements following a
left CVA. The PNF pattern that represents the best choice to promote continued
recovery of the right upper extremity through the use of out-of-synergy
movements is:
A. bilateral symmetrical D1 thrust and reverse thrust.
B. bilateral symmetrical D2F and D2E, elbows straight.
C. chop, reverse chop with right arm leading.
D. lift, reverse lift with right arm leading.

C26 C3 2
26. A 40 year-old individual has limited endurance as a result of a sedentary lifestyle.
There is no history of cardiorespiratory problems. Following an exercise tolerance
test, which was negative, an appropriate INITIAL exercise prescription for this
individual would be:
A. 30 - 60% HRmax.
B. 40 - 60% HRmax.
C. 45 - 55% of VO2max.
D. 60 - 90% HRmax.

474800226.doc
8 Practice Questions

C27 C3 2
27. Following a mild myocardial infarction, a 53 year-old patient is referred to
physical therapy for exercise conditioning. During an initial exercise session he
reports chest pain, appears anxious, and wants to go back to his room to rest.
After a rest period, your BEST course of action is to:
A. contact his doctor immediately.
B. contact the charge nurse on his floor immediately.
C. let the physical therapist assistant monitor his vital signs and take him back to
his room.
D. take him back to his room yourself and continue to monitor vital signs.

C28 C4 2
28. A 24 year-old patient with scleroderma is being treated with temperature
biofeedback. You will know if this intervention is successful if the:
A. blood pressure increases.
B. heart rate increases.
C. patient tells you she feels looser, more flexible.
D. skin blushes.

C29 C8 2
29. A patient has been referred to P.T. following a fall injury (fractured left hip with
ORIF). Her medical history reveals a diagnosis of early Alzheimer’s disease
(Stage 1). At this stage of Alzheimer’s, the behaviors you would NOT expect to
find are:
A. memory loss with increasing disorientation of time and date.
B. poor judgment and safety awareness.
C. profound aphasia and global deterioration of mental functions.
D. restlessness and episodes of sundowning.

C30 C6 2
30. A patient recovering from stroke with minimal lower extremity weakness and
spasticity is able to walk without an assistive device. You observe that as he
walks he hikes his pelvis on the affected side during the swing phase. Your BEST
intervention is:
A. active exercises: bridging.
B. active exercises: sitting and standing, marching in place.
C. manual pressure applied downward on the pelvis during swing.
D. passive mobilization and stretch to quadriceps.

C31 C7 3
31. You are instructing a 22 year-old patient with traumatic brain injury how to lock
the brakes on his wheelchair. He is right-handed and his right upper extremity is

474800226.doc
Practice Questions 9

more affected than his left. To get the most rapid results, your BEST training
strategy in this case is to:
A. guide his right hand through the locking motions, then his left.
B. have him practice brake locking using both hands together.
C. have him practice locking the brakes first with his left UE, and then his right.
D. verbally talk him through the locking motions using both hands
simultaneously.

C32 C7 3
32. A physical therapist wants to examine the effects of PNF using the technique of
contract-relax on shoulder ROM. A group of 10 patients with adhesive capsulitis
were recruited. A matched group of patients were given straight plane active-
assisted exercise for the same length of time (3 times/week for 6 weeks). In this
study the independent variable is:
A. active-assisted exercise.
B. adhesive capsulitis.
C. PNF contract-relax technique.
D. ROM.

474800226.doc
10 Practice Questions

C33 C3 1
33. Clinical findings in a patient with severe emphysema would NOT include:
A. clubbing.
B. cor pulmonale.
C. cyanosis.
D. decreased A-P to lateral chest ratio.

C34 C1 3
34. A patient is receiving physical therapy after an ACL repair. After 4 weeks of
treatment the patient still complains of pain and instability even though he reports
he has been faithful with his home exercise program and wearing his brace. The
original referral was for 7 to 8 weeks of physical therapy. Your BEST course of
action is to:
A. complete the full 8 weeks of treatment and carefully document his lack of
improvement to ensure full insurance coverage.
B. discontinue the treatment.
C. discontinue the treatment; discuss your findings and refer the patient back to
his referring physician.
D. recommend to the patient that he get a second medical opinion.

C35 C1 2
35. During a postural screen for chronic shoulder pain in a recreational swimmer, you
observe excessive internal/medial rotation of the shoulders and winging of the
scapula during overhead motion. The overall focus of your intervention should
be:
A. strengthening of middle and lower trapezius and stretching of pectoral
muscles.
B. strengthening of pectoral muscles and stretching of upper trapezius.
C. strengthening of rhomboids and stretching of upper trapezius.
D. strengthening of upper trapezius and stretching of pectoral muscles.

C36 C7 3
36. When billing Medicare, Medicaid and many other third party payers, providers
are required to use the appropriate:
A. Common Procedural Coding System of Center for Medicare & Medicaid
Services (CMS).
B. CPT and Common Procedural Coding System procedure codes.
C. Current Procedural Terminology (CPT) procedure codes.
D. Resource Based Relative Value Scale (RBRVS).

C37 C1 2
37. The spinal defect shown in the diagram should be managed with avoidance of
lumbar spinal:

474800226.doc
Practice Questions 11

A. extension.
B. flexion.
C. lateral flexion.
D. rotation.

C38 C2 2
38. A 32 year-old computer specialist is unable to work because of weakness and
altered sensation in her dominant right hand. She complains of pain and tingling
of the thumb, index finger, long finger, and radial half of the ring finger. You
observe thenar weakness and atrophy. Strength, reflexes, and sensation are within
normal limits throughout the remainder of the right upper extremity. Her signs
and symptoms are characteristic of:
A. carpal tunnel syndrome.
B. cervical root compression.
C. pronator teres syndrome.
D. ulnar nerve compression.

474800226.doc
12 Practice Questions

C39 C2 2
39. A 34 year-old presents with symmetrical weakness that had affected the distal
lower extremity muscles first but now has ascended to include proximal, trunk
and upper extremity muscles. The motor segments of the lower cranial nerves are
also showing deficits. He complains of abnormal sensations of tingling and
burning of the affected extremities. Consciousness, cognition, and communication
are all normal. His signs and symptoms are characteristic of:
A. amyotrophic lateral sclerosis.
B. Guillain-Barré syndrome.
C. multiple sclerosis.
D. post-polio syndrome.

C40 C3 3
40. A patient recovering from surgery for triple coronary artery bypass grafts is
scheduled to begin a Phase III cardiac rehabilitation program. During the
resistance training portion of the circuit training protocol, you instruct her to
AVOID Valsalva’s maneuver because:
A. a cholinergic or vagal response can occur.
B. heart rate and blood pressure are elevated.
C. slowing of pulse and increased venous pressure is expected.
D. the decreased return of blood to the heart can lead to pitting edema.

C41 C4 2
41. A patient experiences color changes in the skin during position changes of the
foot. During elevation, pallor develops. When the limb is then positioned in the
seated hanging position, hyperemia develops. These changes are indicative of:
A. arterial insufficiency.
B. chronic venous insufficiency.
C. deep vein thrombophlebitis.
D. lymphedema.

C42 C8 2
42. A 73 year-old is referred to physical therapy for an examination of balance. He
has a recent history of falls (two in the last 6 months). Based on your knowledge
of balance changes in the elderly and scoring of standardized balance measures,
the test data that BEST indicates increased fall risk is:
A. a score on the Tinetti Performance Oriented Mobility Assessment (POMA) of
27.
B. A Timed Get Up & Go test result of 13 seconds.
C. Berg Balance score of 50.
D. Functional Reach of 7 inches.

C43 C6 3

474800226.doc
Practice Questions 13

43. A patient walks with a Trendelenburg gait. The MOST appropriate intervention to
correct this problem is:
A. bridging, holding with Theraband around both thighs.
B. half kneeling, weight shifting onto the weak side (foot).
C. standing, stepping with the weaker limb, forward and backward.
D. supine, lateral leg slides.

C44 C3 2
44. A 72 year-old patient is walking on a treadmill in the physical therapy department
while his vital signs are being monitored. It is noted that his SaO2 drops from
97% to 95%. In this case, it would be BEST to:
A. not use supplemental O2.
B. place 2 liters of O2 by nasal cannula on the patient for the remainder of the
exercise session.
C. place a 100% O2 face mask on the patient for the remainder of the exercise
session.
D. place a 40% O2 face mask on the patient for the remainder of the exercise
session.

474800226.doc
14 Practice Questions

C45 C8 2
45. You are evaluating the needs of a 6 year-old child who is diagnosed with
myelodysplasia at the T10 level. You determine the most appropriate mobility
device for this child to use in the school environment is a:
A. bilateral HKAFO.
B. bilateral KAFO.
C. lightweight wheelchair.
D. parapodium.

C46 C1 3
46. A patient has limited right rotation caused by left thoracic facet joint capsular
tightness. The intervention that would best facilitate improved right rotation in
sitting is:
A. trunk extension with left rotation.
B. trunk extension with right rotation.
C. trunk flexion with left rotation.
D. trunk flexion with right rotation.

C47 C1 1
47. EMG activity in the lower extremities during erect standing is continuous in the :
A. anterior tibialis and peroneals.
B. posterior tibialis and intrinsic foot muscles.
C. quadriceps femoris and anterior tibialis.
D. soleus and gastrocnemius.

C48 C7 3
48. A patient had a fall while walking in the parallel bars. The incident report of the
event should include:
A. the cause of the incident, the corrective actions taken, names of those
involved.
B. the cause of the occurrence, the name of the injured and the date of the
occurrence.
C. the name of those involved, witnesses, what occurred when it occurred, where
it occurred only in event of an injury.
D. the name of those involved, witnesses, what occurred, when it occurred, where
it occurred.

C49 C5 3
49. Four days ago, your patient sustained a deep contusion of the right lateral thigh as
a result of a blow on the leg by a steel beam. Following several cryotherapy
treatments, you choose to apply ultrasound. The US parameters of choice are:
A. continuous US at 1 MHz.
B. continuous US at 3 MHz.

474800226.doc
Practice Questions 15

C. pulsed US at 1 MHz.
D. pulsed US at 3 MHz.

C50 C2 2
50. A 99 year-old woman was found unconscious at home. Two days later you
examine her in the hospital. Your findings include normal sensation and
movement on the right side of the body with impaired sensation (touch, pressure,
proprioception) and paralysis on the left side of the body. The left side of her
lower face and her trunk are similarly impaired. The MOST LIKELY location of
the lesion is the:
A. cerebral cortex: left parietal lobe.
B. cerebral cortex: right parietal lobe.
C. left side of the brainstem.
D. spinal cord.

C51 C2 1
51. The loss of sensory function in peripheral neuropathy is often among the first
noticeable symptoms. If more than one nerve is involved, the sensory loss
typically appears as:
A. allodynia of the feet accompanied by pronounced dorsiflexor weakness.
B. bandlike dysesthesias and paresthesias in the hips and thighs.
C. paresthesias affecting primarily the proximal limb segments and trunk.
D. stocking and glove distribution.

474800226.doc
16 Practice Questions

C52 C3 2
52. You are supervising a Phase II cardiac rehabilitation class of 10 patients. One of
the patients, who is being monitored with radiotelemetry, is having difficulty.
You decide to terminate this patient’s exercise session and alter his prescription to
reduce the exercise intensity when you observe:
A. 1 mm ST-segment depression, upsloping.
B. a 20 AV heart block.
C. an increase in HR 20 BPM above resting.
D. an increase in systolic BP to 150 and diastolic BP to 90.

C53 C4 3
53. A 51 year-old patient presents with severe claudication which is evident when he
walks distances greater than 200 feet. He also exhibits muscle fatigue and
cramping of both calf muscles. Upon examination, you find his skin is pale and
shiny with some trophic nail changes. Your BEST choice for intervention is to:
A. avoid any exercise stress until he has been on calcium channel blockers for at
least 2 weeks.
B. begin with an interval walking program, exercising only to the point of pain.
C. utilize a walking program of moderate intensity, instructing the patient that
some pain is expected and to be tolerated.
D. utilize nonweightbearing exercises such as cycle ergometry.

C54 C8 3
54. A 72 year-old is hospitalized with diabetes and a large stage II plantar ulcer
located over his right heel. He has been non-weightbearing for the past 2 weeks as
a result of the ulcer. Based on your knowledge of this condition the BEST
intervention is:
A. clean and bandage with a dry sterile dressing; provide custom fitted, pressure
relieving orthotic shoe insert.
B. clean and debride the wound in a whirlpool followed by a hydrogel dressing
and pressure relief.
C. request a surgical consult; physical agents will not promote healing.
D. wash the foot and apply skin lubricants; a custom made shoe is indicated.

C55 C6 3
55. You are prescribing a wheelchair for a patient with left hemiplegia. The MOST
appropriate feature to include in this prescription:
A. are detachable arm rests
B. are elevating legrests.
C. is a 17 inch seat height.
D. is a 20 inch seat height.

C56 C6 3

474800226.doc
Practice Questions 17

56. A patient presents with decreased strength, 3/5, in the right quadriceps. During
gait, you expect his greatest difficulty will occur at:
A. heel off.
B. heel strike.
C. midstance.
D. terminal swing.

C57 C1 2
57. A patient who was casted for 3 weeks following a Grade III right ankle sprain has
been referred to physical therapy for mobility exercises. Examination shows a
loss of 10 degrees of dorsiflexion. The patient will have the MOST difficulty in:
A. ambulating barefoot.
B. ambulating over rough surfaces.
C. descending a ramp.
D. descending stairs.

474800226.doc
18 Practice Questions

C58 C6 3
58. The torque output produced during isokinetic exercise involving the hamstrings in
the sitting position is:
A. higher due to eccentric assistance of the quadriceps.
B. higher than the torque actually generated by the contracting hamstrings.
C. lower due to resistance of the quadriceps.
D. lower than the torque actually generated by the hamstrings.

C59 C1 3
59. A manual therapy technique utilized to correct a closing restriction of T5 on T6
is:
A. central P/A pressure at a 45 degree angle on the spinous process of T5 while
stabilizing T6.
B. central P/A pressure at a 60 degree angle on the spinous process of T6 while
stabilizing T5.
C. unilateral P/A pressure at a 45 degree angle on the right transverse process of
T6 while stabilizing T5.
D. unilateral P/A pressure at a 60 degree angle on the left transverse process of
T6 while stabilizing T5.

C60 C2 2
60. A patient presents with an acute onset of vertigo over night. Symptoms worsen
with rapid change in head position. If the head is held still, symptoms subside
usually within 1 to 2 minutes. The MOST likely cause of this patient’s problem
is:
A. acoustic neuroma.
B. benign paroxysmal positional vertigo.
C. bilateral vestibular neuritis.
D. Ménière’s disease.

C61 C3 2
61. A patient with a significant history for coronary artery disease tells you he is
currently taking atropine. Based on your knowledge of the effects of this
medication you expect:
A. bradycardia at rest and with exercise.
B. increased heart rate and contractility.
C. reduced blood pressure at rest and with exercise.
D. reduced myocardial ischemia and heart rate.

C62 C4 2
62. A 67 year-old patient recovering from stroke is on warfarin (Coumadin). During
his rehabilitation, it would be important to watch for:
A. cellulitis and xeroderma.

474800226.doc
Practice Questions 19

B. edema and dermatitis.


C. hematuria and ecchymosis.
D. palpitations and edema.

C63 C8 3
63. A 91 year-old patient has reduced vision as a result of bilateral cataracts. Which
of the following is NOT an appropriate intervention for this patient?
A. avoiding having him walk on shiny floor surfaces.
B. highlighting steps with pastels, blues and greens.
C. minimizing visual distractions in his immediate environment.
D. using high illumination for reading and ADLs.

C64 C6 2
64. A patient is having difficulty with stair climbing. He is able to position his foot on
the step but is unable to transfer the weight of his body up to the next stair level.
The PRIMARY muscle that is responsible for elevating the body is the:
A. gastrocnemius-soleus.
B. gluteus maximus.
C. quadriceps femoris.
D. tibialis anterior.

474800226.doc
20 Practice Questions

C65 C4 2
65. A 72 year-old patient has been hospitalized, on complete bedrest, for 10 days.
Your referral requests mobilization out-of-bed and ambulation. The patient tells
you that today his right calf is aching. If he gets up and moves around he is sure
he will feel better. Your examination reveals calf tenderness. You decide to:
A. ambulate the patient with elastic stockings.
B. begin with ankle pump exercises in bed.
C. begin with sitting the patient up over the edge of the bed for 5 minutes.
D. postpone ambulation and report your findings immediately.

C66 C8 1
66. A nine year-old boy with Duchenne’s muscular dystrophy is a new patient
through homecare. You should BEGIN your examination by:
A. asking the child and his parent/caretakers to describe the boy’s most serious
physical problems from their perspectives.
B. asking the parents to outline the child’s developmental milestones.
C. performing a complete motor examination.
D. performing a functional examination using the weeFIM.

C67 C1 1
67. During an examination of an adolescent female who complains of anterior knee
pain, you observe that the lower extremity shows medial femoral torsion and
toeing-in position of the feet. The lower extremity position may be indicative of
excessive hip:
A. anteversion.
B. lateral/external rotation.
C. medial/internal rotation.
D. retroversion.

C68 C1 3
68. A patient with osteoporosis and no fractures complains of increased mid and low
back pain during breathing and most other activities. The MOST appropriate
interventions for this patient would include patient education and:
A. trunk extension and abdominal stabilization exercises.
B. trunk flexion and extension exercises.
C. trunk flexion, extension, and rotation exercises.
D. trunk rotation and abdominal stabilization exercises.

C69 C1 2
69. A single, 22 year-old female, who is 3 months pregnant, walks into your facility.
She complains of shoulder and leg pain. She has a black eye and some bruising at
the wrists. The state in which you practice has direct access. An appropriate
course of action is:

474800226.doc
Practice Questions 21

A. administer massage for bruising, TENs and ice modalities for pain, as
indicated by the examination findings.
B. direct the patient to the nearest Ambulatory Care Center for physician
evaluation.
C. do a comprehensive examination and send her to the emergency room.
D. do a comprehensive examination, and if you suspect abuse report your
findings to the appropriate authorities.

C70 C5 3
70. A 73 year-old patient presents with a stage III decubitus ulcer on the plantar
surface of the right foot. After a series of conservative interventions with limited
success, the therapist chooses to apply electrical stimulation for tissue repair. The
electrical current BEST suited in this case is:
A. high volt monophasic pulsed current.
B. interferential current.
C. low volt biphasic pulsed current.
D. Russian current.

474800226.doc
22 Practice Questions

C71 C5 3
71. A patient presents with pain radiating down the posterior hip and thigh as a result
of a herniated disk in the lumbar spine. You decide to apply mechanical traction.
If the patient can tolerate it, the PREFERRED patient position is:
A. prone with no pillow.
B. prone with pillow under the abdomen.
C. supine with both knees flexed.
D. supine with one knee flexed.

C72 C2 3
72. A patient recovering from stroke demonstrates hemiparesis of his right upper
extremity and moderate flexion and extension synergies (flexion stronger than
extension). Your goal is strengthen the shoulder muscles first, specifically the
deltoid. Your BEST choice is to promote:
A. abduction with elbow extension.
B. abduction with elbow flexion.
C. horizontal adduction with elbow extension.
D. horizontal adduction with elbow flexion.

C73 C2 2
73. A patient recovering from a middle cerebral artery stroke presents with gaze
deviation of the eyes. In this type of stroke the involved eye may deviate toward:
A. down and out.
B. the hemiplegic side.
C. the sound side.
D. up and in.

C74 C3 2
74. A 44 year-old patient is referred for physical therapy following an exercise
tolerance test. His physician reports the test was positive and had to be terminated
at 7 minutes. Based on your knowledge of this procedure, you expect the patient
may have exhibited:
A. a hypertensive response, BPs of at least 170/95.
B. ECG changes from baseline (1 mm ST-segment elevation).
C. ECG changes from baseline (3 mm horizontal or downsloping, ST-segment
depression).
D. increasing angina and dyspnea with progressive increases in the treadmill
speed and grade.

C75 C1 2
75. A twenty-one year old college soccer player sustained a hyperextension knee
injury when kicking the ball with his other lower extremity. The patient was taken
to the emergency room of a local hospital and was diagnosed with “knee sprain”.

474800226.doc
Practice Questions 23

He was sent to physical therapy the next day for aggressive rehabilitation. As part
of the examination to determine the type of treatment plan to implement, the
therapist conducted the anterior drawer test. The type of exercise that is
contraindicated in the acute phase of treatment if a positive test is found includes:
A. agility training.
B. closed-chain terminal knee extension exercises.
C. open-chain terminal knee extension exercises.
D. short arc quad exercises.

C76 C4 3
76. A patient with a grade III diabetic ulcer is being treated with a calcium alginate
wound dressing. This type of dressing can be expected to:
A. absorb exudate and allow rapid moisture evaporation.
B. facilitate autolytic debridement.
C. prove impermeable to bacteria.
D. provide semirigid support for the limb.

474800226.doc
24 Practice Questions

C77 C4 3
77. You receive a referral to treat a patient with cirrhosis and Hepatitis B. As a result
of these diagnoses, you should:
A. ask the patient to refrain from touching you without gloves.
B. avoid all direct physical contact with the patient.
C. avoid direct exposure to blood and body fluids.
D. treat the patient as you would any other patient; no precautions are necessary.

C78 C6 2
78. A patient with a transtibial amputation is learning to walk using a PTB prosthesis.
He is having difficulty maintaining prosthetic stability from heel-strike to footflat.
The muscle group that is MOST LIKELY weak are the:
A. back extensors.
B. hip flexors.
C. knee extensors.
D. knee flexors.

C79 C3 2
79. A patient with a history of coronary artery disease and recent myocardial
infarction is exercising in the P.T. gym while on an ECG telemetry monitor. You
observe 5 consecutive PVCs on her ECG. Normal sinus rhythm then returns.
Your BEST course of action is to:
A. activate the emergency medical response team.
B. have the patient sit down, continue monitoring, and notify the physician.
C. stop the exercise and send her back to her room as soon as possible.
D. stop the exercise, have her rest, then resume at a lower intensity.

C80 C7 3
80. A 16 year-old patient recovering from traumatic brain injury is unable to bring
her right foot up on the stair during stair climbing training. The BEST training
activity is to:
A. have her practice marching in place.
B. passively bring her foot up and place it on the 7 inch step.
C. practice stair climbing inside the parallel bars using a 3 inch step.
D. strengthen her hip flexors using an isokinetic training device before attempting
stair climbing.

C81 C3 3
81. Which of the following interventions is NOT beneficial for a patient with a right
lower lobe viral pneumonia:
A. ambulation activity, monitoring SaO2, HR, RR, and BP.
B. breathing exercises, encouraging right lateral costal expansion.

474800226.doc
Practice Questions 25

C. postural drainage, percussion, and shaking to the right lower lobe and other
areas with abnormal auscultatory findings.
D. teaching an independent exercise program to the patient.

C82 C4 2
82. A patient suffered a spinal cord injury with a complete injury (ASIA Level A) at
T10. It is now three months post-injury and she is refusing to participate in her
functional training program because the major focus is wheelchair independence.
She is sure she is going to walk again. Your BEST approach is to:
A. discuss the harmful effects of denial and restrict all discussions to promoting
wheelchair independence.
B. recognize the patient’s need for hope while outlining realistic short term goals
to improve independence.
C. refer the patient for psychological counseling and discharge her from P.T.
D. send the patient home for a short time so she will recognize the need for
wheelchair training.

474800226.doc
26 Practice Questions

C83 C8 3
83. You are on a home visit, scheduled at lunchtime, visiting an 18 month-old child
with moderate developmental delay. You notice the child and mother are
experiencing difficulties with feeding. The child is slumped down in the highchair
and is unsuccessfully attempting to use a raking grasp to lift cereal pieces to her
mouth. Both the child and mother are frustrated. Your FIRST intervention should
be to:
A. position the child in a proper sitting position using postural supports.
B. recommend the mother feed the child baby food instead of cereal for a few
more months.
C. recommend the mother return to breast feeding for a few more months.
D. work on desensitizing the gag reflex.

C84 C1 3
84. A 55 year-old woman was sent to physical therapy with a diagnosis of “frozen
shoulder”. The MOST effective mobilization technique for restricted shoulder
abduction is:
A. inferior glide at 55 degrees of abduction.
B. inferior glide at 95 degrees of abduction.
C. lateral glide in neutral position.
D. posterior glide at 10 degrees of abduction.

C85 C1 2
85. A male patient presents with insidious onset of pain in the jaw that is referred to
the head and neck regions. As best as he can recall, it may be related to biting into
something hard. Cervical ROM is limited in flexion by 20 degrees, cervical
lateral flexion limited to the left by 10 degrees. Mandibular depression is 10mm
with deviation to the left, protrusion is 4mm, and lateral deviation is 15mm to
right and 6mm to left. Based on these findings the diagnosis for this patient would
be:
A. capsule-ligamentous pattern of TMJ on the left.
B. cervical spine and TMJ capsular restrictions on the left.
C. weak lateral pterygoids on the left.
D. weak lateral pterygoids on the right.

C86 C7 3
86. Your patient, a 35 year-old administrative assistant and mother of three is being
treated for a Colles’ fracture. Her husband wants to look at her medical record. As
her physical therapist you should:
A. deny access to the chart unless written permission by his wife is granted.
B. give him the chart as he is a family member and has a right to view the
information.
C. let him look at the chart with your supervision.

474800226.doc
Practice Questions 27

D. not let him look at the chart because he may misinterpret the documentation.

C87 C3 2
87. A patient with a recent history of rib fractures suddenly becomes short of breath
during secretion removal techniques. The patient looks panicked and complains
of sharp pain in the left chest. A quick screen shows a deviated trachea to the
right among other signs and symptoms. The MOST LIKELY explanation for the
above is:
A. angina.
B. mucous plugging of an airway.
C. pneumothorax.
D. pulmonary emboli.

C88 C5 3
88. Your patient presents with supraspinatus tendinitis. After the initial cryotherapy,
you decide to apply ultrasound. To effectively treat the supraspinatus tendon, you
would place the shoulder joint in:
A. abduction and external rotation.
B. abduction and internal rotation.
C. neutral position and external rotation.
D. neutral position and internal rotation.

474800226.doc
28 Practice Questions

C89 C2 3
89. A patient is 5 weeks poststroke and is demonstrating good recovery of her right
upper extremity, characterized as stage 4 recovery stage. Your BEST choice for a
training activity is to have the patient:
A. bear weight on the extended right arm in sitting.
B. put on socks and shoes while in sitting.
C. reach forward to bear weight with the right arm extended against the wall.
D. reach overhead with right arm straight.

C90 C2 2
90. A patient is 2 days post left CVA and has just been moved from the intensive care
unit to a stroke unit. When you arrive to begin your examination, you find his
speech slow and hesitant. He is limited to one and two-word productions and his
expressions are awkward and arduous. His writing skills parallel his speech
difficulties. However, he demonstrates good comprehension. His difficulties are
consistent with:
A. Broca’s aphasia.
B. dysarthria.
C. global aphasia.
D. Wernicke’s aphasia.

C91 C2 3
91. A 17 year-old patient recovering from traumatic brain injury is functioning at
Stage IV on the Rancho Los Amigos Levels of Cognitive Functioning Scale.
During your initial examination she becomes agitated and tries to bite you. Your
BEST course of action is to:
A. engage in a calming activity and observe behaviors closely.
B. postpone the examination for one week and then try again.
C. postpone the examination until later in the day when she calms down.
D. restructure the formal examination so you can complete it in three very short
sessions.

C92 C3 2
92. During an education session with patients and families, you are going over the
major signs and symptoms suggestive of myocardial infarction. You determine
the outcomes of the class have been successful. The participants can successfully
identify major signs and symptoms. Which of the following is NOT an initial
presenting symptom of M.I.?
A. dizziness or syncope upon arising from bed.
B. pain or discomfort in the chest, neck, jaw, or arms.
C. palpitations or tachycardia.
D. unusual fatigue or shortness of breath, clammy skin.

474800226.doc
Practice Questions 29

C93 C3 2
93. A 32 year-old runner is examined in physical therapy for anterior pain in the right
lower leg. Her resting heart rate is found to be 46 bpm. The MOST LIKELY
explanation for this is that:
A. a compensatory response to prolonged endurance activity is depressed heart
rate with increased respiratory rate.
B. a low heart rate is suggestive of a hypotensive disorder.
C. coronary pathology should be suspected with this abnormally low heart rate.
D. prolonged endurance training has resulted in a low heart rate.

C94 C4 2
94. A patient has been taking corticosteroids (hydrocortisone) for management of
adrenocortical insufficiency. She is referred to physical therapy for mobility
training following a prolonged hospitalization. Potential adverse effects that one
can expect from prolonged use of this medication include:
A. confusion and depression.
B. decreased appetite and weight loss.
C. hypotension and myopathy.
D. muscle wasting and increased risk of fracture.

474800226.doc
30 Practice Questions

C95 C4 3
95. A patient is referred for wound debridement using hydrotherapy. Upon
examination, you find the wound is clean, with healthy red granulation tissue and
minimal drainage. Your BEST choice for wound management is to:
A. not use a whirlpool additive.
B. use high-pressure wound lavage while in the whirlpool.
C. use povidone-iodine as a whirlpool additive.
D. use sodium hypochlorite as a whirlpool additive.

C96 C6 3
96. Recently a 10 year-old patient has begun walking with supination of her foot.
With her shoe off, you find a new callus on the lateral side of the metatarsal head
of the 5th toe. Your BEST choice for orthotic prescription is:
A. scaphoid pad.
B. Thomas heel.
C. viscoelastic shoe insert with forefoot lateral wedge.
D. viscoelastic shoe insert with forefoot medial wedge.

C97 C6 3
97. A 74 year-old patient is recovering from a right total hip replacement
(posterolateral incision, cementless fixation). The MOST appropriate type of bed-
to-wheelchair transfer to teach is to have the patient:
A. increase forward flexion during standing-up.
B. transfer to the operated side.
C. transfer to the sound side.
D. use a transfer board.

C98 C6 3
98. A patient with paraplegia at the T10 level wants to participate in wheelchair
basketball. He asks you what options he should look for in a wheelchair. You tell
him it would be important to include:
A. a folding frame.
B. a mid-scapular seat back.
C. a rigid frame.
D. hard-rubber tires.

C99 C8 3
99. You are treating a child with mild developmental delay secondary to moderate
prematurity at birth who is just learning to sit. The best choice for training activity
is:
A. prone tilting reactions.
B. sideward protective extension in sitting.
C. standing tilting reactions.

474800226.doc
Practice Questions 31

D. supine tilting reactions.

C100 C3 2
100. A 80 year-old patient with emphysema with no history of cardiac disease
performs a 12 minute exercise tolerance test, covering 1,106 feet. His vital signs
prior to exercise were: HR 104, BP 130/76, SaO2 92%. At peak exercise his vital
signs were: HR 137, BP 162/74, SaO2 92%. To calculate his exercise intensity
parameters, the BEST method would be to use:
A. 40 to 50% of Max METs.
B. 70 to 80% of HRmax.
C. 70 to 85% of age adjusted predicted HRmax.
D. Karvonen’s formula (HR reserve).

C101 C8 2
101. An expected outcome for a fifteen year-old boy with Duchenne’s muscular
dystrophy is:
A. ambulation with a walker.
B. increase strength and range of motion of the lower extremities.
C. independent in ambulation.
D. independent in wheelchair mobility.

474800226.doc
32 Practice Questions

C102 C1 1
102. When performing scoliosis screening in a school setting, the optimal age to screen
for girls is:
A. 15-17.
B. 6-8.
C. 9-11.
D. 12-14.

C103 C1 3
103. A patient with a confirmed left C6 nerve root compression due to foraminal
encroachment complains of pain in his left thumb and index finger. The MOST
effective cervical position to alleviate this radicular pain in weightbearing is:
A. left sidebending.
B. lower cervical extension.
C. lower cervical flexion.
D. right rotation.

C104 C1 1
104. A patient is standing with excessive subtalar pronation. Possible correlated
motions or postures are:
A. femoral external rotation, and pelvic external rotation.
B. femoral internal rotation, and pelvic external rotation.
C. tibial, femoral, and pelvic external rotation.
D. tibial, femoral, and pelvic internal rotation.

C105 C7 3
105. When performing a chart audit, you realize a date of service was documented
inappropriately. You should:
A. do nothing.
B. put a single line through the incorrect date, initial it, document the correct
date, and the date the correction was made.
C. use “white out” and put the correct date of service in.
D. write over the date of service to correct it with ink.

C106 C5 3
106. You are applying high volt pulsed current to the vastus medialis to improve
patellar tracking during knee extension. Your patient complains that the current is
uncomfortable. To make the current more tolerable to the patient, yet maintain a
good therapeutic effect, you should consider adjusting the:
A. current intensity.
B. current polarity.
C. pulse duration.
D. pulse rate.

474800226.doc
Practice Questions 33

C107 C2 2
107. A patient has a 10 year history of multiple sclerosis and presents with drooping of
the right upper eyelid, constriction of the pupil, and vasodilation with absence of
sweating on the face and neck. These signs are characteristic of:
A. Argyll Robertson pupil.
B. homonymous hemianopsia.
C. Horner’s syndrome.
D. nystagmus.

C108 C2 2
108. A patient presents with symptoms of uncoordinated eye movements and profound
gait and trunk ataxia. He has difficulty with postural orientation to vertical and
tends to tip over even if his eyes are open. Examination of the extremities reveals
little change in tone or coordination. You suspect involvement of the:
A. basal ganglia.
B. premotor cortex.
C. spinocerebellum.
D. vestibulocerebellum.

474800226.doc
34 Practice Questions

C109 C3 2
109. A 37 year-old patient experiences cramping, pain, and fatigue of the buttock after
walking 400 feet. When he stops exercising, the pain goes away immediately.
You check his medical record and find no mention of this problem. Your BEST
course of action is to:
A. advise the patient he will have to curtail his walking until you do further tests.
B. prescribe an intermittent walking program with enough rests to eliminate the
pain.
C. provide the patient with a home exercise program of spinal extension
exercises.
D. report this finding to the physician as it might be indicative of peripheral
vascular disease.

C110 C4 2
110. A 42 year-old patient has marked elevation of blood pressure. He complains of
mild to severe midabdominal pain that increases upon exertion. Palpation reveals
a pulsing mass in the lower abdomen. You should:
A. discontinue treatment and notify his physician immediately.
B. do not treat; instruct the patient to see his physician as soon as possible.
C. instruct in relaxation exercises; postpone all other treatment.
D. provide warm hot packs to the abdomen to relieve the pain; do not exercise.

C111 C4 3
111. A patient with a small, purulent wound located on his left heel is referred for
wound irrigation. It would be BEST to:
A. irrigate the wound in the whirlpool using lukewarm water.
B. irrigate the wound with hydrogen peroxide while having the patient wear
protective garments.
C. irrigate the wound with povidone-iodine while wearing protective eyewear,
garment, and gloves.
D. protect the wound borders with protective shields to prevent pulsating lavage
from damaging healthy tissue.

C112 C8 2
112. An 85 year-old female wheelchair dependent resident of a community nursing
home has a diagnosis of organic brain syndrome, Alzheimer’s type. In developing
her plan of care, it is important to understand that she:
A. can usually be trusted to be responsible for her daily care needs.
B. can usually be trusted with transfers with appropriate positioning of the
wheelchair.
C. is more likely to remember current experiences than past ones.
D. will likely be resistant to activity training if unfamiliar activities are used.

474800226.doc
Practice Questions 35

C113 C6 3
113. To increase the stride length of a patient with a right transfemoral amputation
who uses a total contact prosthesis, you should:
A. facilitate the gluteals.
B. provide anterior directed resistance to the right PSIS during swing.
C. provide posterior directed resistance to the left ASIS during swing.
D. provide posterior directed resistance to the right ASIS during stance.

C114 C6 3
114. A patient with a complete C7 spinal cord injury is having difficulty with pushups
while in his wheelchair. The MOST appropriate lead-up activity to enhance
wheelchair pushups is:
A. prone-on-elbow pushups.
B. shoulder shrugs.
C. supine bench press using 50% one repetition max.
D. supine-on-elbows pushups.

C115 C7 3
115. You are concerned that a student you are supervising, who is on a final clinical
rotation, is not exhibiting appropriate behaviors. Specifically, he does not seem to
be willing to listen to others or demonstrate tolerance and sensitivity to patient
needs. The MOST appropriate conclusion to reach from these behaviors is:
A. affective objectives dealing with attending are not being met.
B. he does not value respect for human dignity.
C. his problem is one of conceptualization and forming appropriate judgments.
D. his problem is one of failure to accept responsibility.

474800226.doc
36 Practice Questions

C116 C7 3
116. A therapist wants to investigate the effectiveness of use of the therapeutic pool for
decreasing pain in a group of patients with fibromyalgia. Two groups of patients
were recruited. One group was assigned to exercises and walking in the pool 3
times/week for 6 months. The other group was assigned to a gym walking
program for the same amount of time. At the end of the study, outcomes were
assessed using the McGill Pain Questionnaire and the Health Status
Questionnaire. In order to improve reliability, the lead investigator should:
A. have another therapist reassess after 6 months and compare to normalized
scores.
B. have the same therapist reassess the patients after 6 months.
C. perform all the final assessments himself and compare to the initial
assessments performed by a core group of therapists.
D. utilize a core of 4 experienced therapists to randomly complete all the
assessments.

C117 C3 2
117. Your patient is a 55 year-old individual who has many risk factors for coronary
artery disease. He is interested in beginning an exercise program to improve his
cardiac health. The most accurate measure of exercise intensity to monitor during
his first exercise session is:
A. heart rate.
B. MET level.
C. rating of perceived exertion.
D. respiratory rate.

C118 C8 2
118. A mother brings her 8 week-old infant to be examined at Early Intervention
because she noticed that the infant was taking steps in supported standing at two
weeks but was not able to do it now. You should:
A. explain that this is normal and that the stepping was a newborn reflex which
has gone away.
B. recommend that a full developmental exam be performed by the Early
Intervention team.
C. recommend that the mother bring the infant to a pediatric neurologist.
D. tell her she must have been mistaken.

C119 C1 1
119. A patient complains of pain with mouth opening that makes it difficult for her to
eat foods that require chewing. Mouth opening was found to be within normal
limits of:
A. 15-24mm.
B. 25-34mm.

474800226.doc
Practice Questions 37

C. 35-49mm.
D. 50-65mm.

C120 C1 3
120. Correction of flexible forefoot varus with excessive subtalar pronation is
accomplished by a customized orthosis with:
A. lateral forefoot posting.
B. medial forefoot and rearfoot varus posting.
C. medial forefoot posting.
D. rearfoot varus posting.

C121 C1 2
121. Your patient, a 40 year-old female, presents with complaints of tingling and
paresthesias in the median nerve distribution of the right forearm and hand. The
following tests were found negative bilaterally: Adson, hyperabduction,
costoclavicular, Phalen’s, and the ulnar nerve Tinel sign. Based on this
information, the diagnosis that has NOT been ruled out is:
A. carpal tunnel syndrome.
B. pronator teres syndrome.
C. thoracic outlet syndrome.
D. ulnar nerve entrapment.

474800226.doc
38 Practice Questions

C122 C7 3
122. A female patient scheduled for a 30-minute treatment session arrives 10 minutes
late. The subsequent patients are also scheduled for 30 minutes sessions and there
is no break in your schedule to accommodate for the patient’s tardiness. You
should:
A. ask the patient if there is a more convenient time for her appointment so she
can benefit from the complete 30-minute session.
B. determine a better treatment time, treat the patient, and bill for the 20 minute
session given.
C. have the office secretary send the patient home until her next scheduled
appointment, with a request to be punctual.
D. treat the patient and bill for 30 minutes of scheduled treatment time.

C123 C5 3
123. A patient has been referred to you s/p fracture of the femur six months ago. The
cast was removed, but the patient is unable to volitionally contract the quadriceps.
You decide to apply electrical stimulation to stimulate the strengthening of the
quadriceps muscle. Your choice of electrode size and placement would consist of:
A. large electrodes, closely spaced.
B. large electrodes, widely spaced.
C. small electrodes, closely spaced.
D. small electrodes, widely spaced.

C124 C2 3
124. A 65 year-old patient is recovering from a right CVA. Due to a series of medical
complications she is still bedridden 6 days post stroke. You want to reduce the
expected negative effects of developing spasticity. The BEST choice of bed
position for this patient is:
A. sidelying on the affected side, with the affected shoulder positioned directly
underneath, the hip slightly extended with knee flexed on a pillow.
B. sidelying on the sound side, affected arm and leg extended at the side, with a
pillow between the knees.
C. supine, trunk in midline with small pillow under the scapula, arm extended on
supporting pillow, and a small towel roll under the knee.
D. supine, trunk in slight lateral flexion to the sound side with elbow flexed and
supported on a pillow, leg straight.

C125 C2 1
125. If the subject’s vision is blocked either by having the subject close the eyes or by
placing a barrier between the part being tested and the subject’s eyes, you can
effectively examine:
A. conscious proprioception but not discriminative touch.
B. discriminative touch and fast pain but not proprioception.

474800226.doc
Practice Questions 39

C. somatosensory integrity.
D. vestibular/visual/somatosensory integration.

C126 C3 2
126. You are examining a patient in the coronary intensive care unit. On auscultation
you hear an adventitious S3 heart sound. This finding is indicative of:
A. aortic valve dysfunction.
B. congestive heart failure.
C. pericarditis.
D. pulmonary valve dysfunction.

C127 C3 2
127. You are advising a 67 year-old individual who wants to take part in a graduated
conditioning program by joining the “Mall Walkers Club”. Which of the
following is LEAST appropriate when prescribing exercise for the healthy
elderly?
A. an initial conservative approach to reduce characteristic muscle fatigability.
B. focus on low intensity and increased duration of exercise to avoid injury.
C. intensity prescribed by HR reserve method and Ratings of Perceived Exertion.
D. intensity prescribed using maximal age-related HR.

474800226.doc
40 Practice Questions

C128 C4 2
128. A 72 year-old patient recovering from stroke has been using a bilateral exerciser
(UBE) to strengthen muscles in his affected right upper extremity. He is now
experiencing burning pain in his shoulder that worsens when his limb is touched
or moved. He also presents with paresthesias and pitting edema in the dorsum of
the hand. ROM of the wrist and fingers is painful and diminished. Your BEST
course of action is to:
A. discontinue all treatment until pain disappears.
B. discontinue UBE exercise, splint the hand and wrist until pain and swelling
disappear.
C. discontinue UBE exercise; provide massage and active assistive range of
motion for the entire extremity.
D. switch to interval exercise and lower the resistance on the UBE.

C129 C8 2
129. A 74 year-old patient has had two recent falls coming home from Bingo after
dark. Her outside steps are well lit. She’s unsure why she has fallen but tells you
both times she fell just as she came into her house, before she even had a chance
to put her purse down and turn on the inside lights. You suspect a problem with:
A. decreased corneal sensitivity.
B. decreased ocular scanning movements.
C. loss of accommodation and near vision.
D. poor light adaptation.

C130 C6 3
130. A patient presents with 2/5 muscle strength in both lower extremities and 3/5
strength in the upper extremities. The MOST appropriate transfer to teach this
patient to move from bed to wheelchair is:
A. dependent 1-man squat transfer.
B. sliding board.
C. stand pivot.
D. stand-by assist.

C131 C6 3
131. A patient with a spinal cord injury at the level of T1 is in the community phase of
his mobility training. In order for him to navigate a standard height curb with his
wheelchair, you tell him to:
A. ascend backwards with the large wheels first.
B. descend backward with the trunk upright and arms hooked around the push
handles.
C. lift the front casters and ascend in a wheelie position.
D. place the front casters down first during descent.

474800226.doc
Practice Questions 41

C132 C3 2
132. A patient with a long history of systemic steroid use for asthma control has a
contraindication for percussion if there is evidence of:
A. ankle edema from fluid retention.
B. BP > 140/90.
C. decreased bone density.
D. muscle wasting.

C133 C1 2
133. A female patient presents with a rapid onset of severe weakness of all small
muscles of the hand, sharp pleuritic pain in the shoulder and subscapular area, and
a hoarse voice for the past three weeks. She is a hair stylist, has a smoking history
of 20 years, is not on any medications, and has not been ill. Her referral states
examine and treat. Based on the above information this patient is MOST LIKELY
exhibiting symptoms of:
A. a Pancoast tumor.
B. carpal tunnel syndrome.
C. pronator teres syndrome.
D. thoracic outlet syndrome.

474800226.doc
42 Practice Questions

C134 C1 2
134. An examination of a 30 year-old male patient reveals the following shoulder signs
and symptoms: excessive AROM and PROM; pain with activity, and on
palpation; normal resisted isometric contractions; a positive load/shift test; and
negative X-ray findings. The MOST LIKELY diagnosis is:
A. atraumatic shoulder instability.
B. impingement.
C. rotator cuff lesion.
D. traumatic anterior shoulder dislocation.

C135 C5 3
135. You are applying cervical traction using a cervical harness. Your patient
complains of pain in the temporomandibular joint during the treatment. You
should consider:
A. decreasing the treatment time.
B. discontinuing traction.
C. readjusting the harness and continuing with the treatment.
D. reducing the traction poundage and continuing with the treatment.

C136 C6 2
136. You are providing physical therapy for a 55 year-old female having a diagnosis of
right shoulder rotator cuff tendonitis. The findings of a worksite ergonomic
assessment indicate that the worker is required to perform repetitive reaching
activities above shoulder height. The most appropriate worksite modification
would be to:
A. allow the worker to take more frequent rest pauses.
B. provide the worker with a taller, sit-stand chair.
C. reposition the height of the shelf to below shoulder height.
D. require the worker to attend a cumulative trauma disorder educational class.

C137 C2 3
137. Your patient has been diagnosed with impingement syndrome of the shoulder.
Following a course of modalities to control pain and inflammation, progression is
to an exercise program to restore normal function of the shoulder. The BEST PNF
diagonal pattern to improve function of the shoulder is:
A. D1 extension.
B. D1 flexion.
C. D2 extension.
D. D2 flexion.

C138 C6 2
138. A 38 year-old patient presents with pain of the right Achilles tendon as well as on
the plantar aspect of the right heel. Pain developed insidiously and has now lasted

474800226.doc
Practice Questions 43

several months. On gait analysis you observe abnormal supination throughout the
stance phase of gait. Your BEST choice for orthotic intervention is a:
A. cushion heel with a rearfoot valgus post.
B. flexible shoe insert with forefoot varus post.
C. metatarsal pad.
D. UCBL insert.

C139 C3 2
139. A patient has a 10 year history of peripheral vascular disease. During auscultation
of the popliteal artery you do NOT expect to find:
A. 1+ pulses.
B. 2+ pulses.
C. a bruit.
D. absence of detectable blood flow.

C140 C4 2
140. The most common infection transmitted to healthcare workers is:
A. hepatitis A.
B. hepatitis B.
C. HIV.
D. tuberculosis.

C141 C4 2
141. A patient presents with pain, joint swelling, positive serum rheumatoid factor, and
increased erythrocyte sedimentation rate. These findings are characteristic of:
A. fibromyalgia.
B. osteoarthritis.
C. rheumatoid arthritis.
D. systemic lupus erythematosus.

C142 C6 1
142. A patient with a transfemoral amputation and an above-knee prosthesis
demonstrates knee instability while standing. His knee buckles easily when he
shifts his weight. You suspect the cause of his problem is a:
A. prosthetic knee set too far anterior to the TKA line.
B. prosthetic knee set too far posterior to the TKA line.
C. tight extension aid.
D. weak gluteus medius.

C143 C6 3
143. A 17 year-old individual with developmental disabilities is referred to your
wheelchair clinic for a new wheelchair. She presents with a severe
kyphoscoliosis. You determine the BEST wheelchair modification to order is a:
A. contoured foam seat.
474800226.doc
44 Practice Questions

B. firm seat back with lateral posture supports and increased seat depth.
C. firm seat with lateral knee positioners.
D. sling seat with dense foam cushion.

C144 C6 3
144. A patient is referred for orthotic gait training after receiving a reciprocating gait
orthosis. In order for this patient to walk correctly, it is important to instruct her in
the correct sequence. She should shift her weight:
A. onto her crutches and swing both legs through together to a position in front of
her crutches.
B. onto her crutches and swing one leg, then the other forward.
C. onto her walker and one leg, tuck her pelvis by extending the upper trunk, and
swing her other leg through.
D. onto her walker, extend the upper trunk, and swing both legs forward together
to approach the walker.

C145 C7 3
145. A 42 year-old patient who is undergoing spinal cord rehabilitation is viewed as
uncooperative by staff. He refuses to complete the training activities you have
outlined for him to promote independent functional mobility. A review of his
history reveals that previously he was the director of his own company, with a
staff of 20. The MOST appropriate strategy you can adopt is to:
A. carefully structure the activities and slow down the pace.
B. have him work with a supervisor since he works best with people in authority.
C. involve him in goal setting and have him participate in structuring the training
session.
D. refer him to another therapist in the hopes that he will have better luck in
engaging the patient.

C146 C7 3
146. A group of 10 patients is recruited into a study investigating the effects of
relaxation training on blood pressure. One group of patients is scheduled to
participate in a cardiac rehabilitation program which includes relaxation training
3 times a week for 12 weeks. The other group of patients is instructed to perform
activities as usual. At the conclusion of the study there was no significant
difference between the groups; BP decreased significantly in both groups. The
investigator can reasonably conclude:
A. both groups had blood pressures initially so high that reductions should have
been expected.
B. cardiac rehabilitation is not effective in reducing blood pressure.
C. the activities of the non-rehab group were not properly monitored and may
account for these results.
D. the rehab group was not properly monitored.

474800226.doc
Practice Questions 45

C147 C6 2
147. A patient is using a right KAFO. During orthotic checkout, you discover the
height of the medial upright is excessive. As she transfers weight to the orthotic
leg during gait, you expect that this patient will demonstrate:
A. anterior trunk bending.
B. lateral lean toward the left.
C. lateral lean toward the right.
D. posterior trunk bending.

C148 C6 2
148. During gait, a patient with hemiparesis drags his toes during swing. Upon further
examination, he has weak dorsiflexors (able to lift the foot against gravity through
1/2 range) and a grade of 2 upon examining tone in his plantar flexors using the
Modified Ashworth Scale. An appropriate orthotic modification to correct this
problem is:
A. a dorsiflexion assist.
B. a dorsiflexion stop.
C. a solid ankle AFO.
D. spiral AFO.

C149 C8 2
149. You are examining a 24 month-old child and you observe that the child can sit
independently, creep in quadruped, pull-to-stand, cruise sideways, but not walk
without support. You conclude that this child is exhibiting:
A. advanced gross motor development.
B. delay in achieving developmental milestones.
C. normal gross motor development.
D. slow maturation that is within normal limits.

C150 C1 3
150. Therapist hand/finger placements for posterior to anterior (PA) mobilization
techniques to improve down-gliding/closure of the T7-8 facet joints should be
located at the:
A. spinous process of T6.
B. spinous process of T8.
C. transverse processes of T7.
D. transverse processes of T8.

C151 C1 2
151. A 40 year-old female complains of waking up several times at night from severe
“pins and needles” in both hands. On awakening, her hands feel numb for half an
hour, and she complains of clumsiness with fine hand movements. Your
examination revealed paresthesias confirmed to medial forearm and hypothenar

474800226.doc
46 Practice Questions

region; reduced grip and pinch strength; and normal tendon reflexes. Based on the
above examination findings your diagnosis is:
A. carpal tunnel syndrome.
B. pronator teres syndrome.
C. thoracic outlet syndrome.
D. ulnar nerve entrapment.

C152 C5 2
152. A 57 year-old auto mechanic has been referred to you with a diagnosis of
degenerative joint disease affecting C2 and C3. The patient complains of pain and
stiffness in the cervical region and transient dizziness with some cervical motions.
Your INITIAL examination procedure should be:
A. a manual muscle test.
B. a vertebral artery test.
C. Adson’s maneuver.
D. Oppenheim test.

474800226.doc
Practice Questions 47

C153 C5 2
153. A 45 year-old patient is referred to PT for mechanical compression. The patient
has a history of recent DVT and now presents with edema of the right lower
extremity, which is very painful, and warm to the touch. You should:
A. apply intermittent compression with cold.
B. apply mechanical compression for 2 hours or more.
C. apply mechanical compression for less than 2 hours.
D. not apply mechanical compression and communicate with the referring
physician.

C154 C2 1
154. To examine a patient with a suspected deficit in graphesthesia, you would ask the
patient, with eyes shut, to identify:
A. a series of letters traced on the hand.
B. different objects placed in the hand and manipulated.
C. different weighted, identically shaped cylinders placed in the hand.
D. the vibrations of a tuning fork when placed on a bony prominence.

C155 C2 3
155. A patient recovering from stroke is ambulatory without an assistive device. He
demonstrates a consistent problem with an elevated and retracted pelvis on the
affected side. The BEST therapeutic exercise strategy is to manually apply:
A. anterior directed pressure during swing.
B. downward compression during stance.
C. light resistance to forward pelvic rotation during swing.
D. resistance to posterior pelvic elevation during swing.

C156 C2 2
156. You suspect lower brainstem involvement in a patient with amyotrophic lateral
sclerosis. You examine the patient and find motor impairments of the tongue with
ipsilateral wasting and deviation on protrusion. These findings confirm
involvement of cranial nerve:
A. IX.
B. X.
C. XI.
D. XII.

C157 C3 2
157. A patient is in the intensive care unit following myocardial infarction. Upon
examination of the ECG tracings in the medical record you observe the following
changes: abnormal Q wave with ST elevation and T wave inversion in leads II,
III, and AVF. The probable location of the infarct is the:
A. anterior wall.

474800226.doc
48 Practice Questions

B. inferior wall.
C. lateral wall.
D. posterior wall.

C158 C4 1
158. A 22 year-old patient is hospitalized in the ICU with extensive trauma following a
motor vehicle accident. A review of her medical record reveals the following lab
values: hematocrit 28%, hemoglobin 10 g/100ml, and serum WBC 12,000/mm3.
The MOST appropriate conclusion the therapist can reach is:
A. all values are abnormal.
B. hematocrit and hemoglobin values are abnormal; WBC is normal.
C. only hematocrit values are abnormal.
D. only serum WBC is abnormal.

474800226.doc
Practice Questions 49

C159 C4 2
159. A patient with a transfemoral amputation is unable to wear his total contact
prosthesis for the past 4 days. Examination of the residual limb reveals erythema
and edema extending over most of the lower anterior limb. He tells you his limb
is very itchy and painful after he scratches it. The MOST LIKELY cause of his
symptoms is:
A. cellulitis.
B. dermatitis.
C. herpes zoster.
D. impetigo.

C160 C4 3
160. A patient is largely confined to bed and has a stage IV sacral pressure ulcer of
three months duration. The BEST choice of intervention is:
A. a two-inch, convoluted foam mattress.
B. gentle wound cleansing and wet-to-dry gauze dressings.
C. nutritional supplements and pressure relief with a flotation mattress.
D. surgical repair.

C161 C6 2
161. Following a hip fracture that is now healed, a patient presents with weak hip
flexors (2/5). All other muscles are within functional limits. During gait, you
expect that she may walk with:
A. backward trunk lean.
B. circumduction.
C. excessive hip flexion.
D. forward trunk lean.

C162 C6 3
162. A 72 year-old patient with a left transfemoral amputation complains that his left
foot is cramping and when he sits it feels all twisted under him. Your BEST
choice of intervention is:
A. appropriate bed positioning with the residual limb in extension.
B. hot packs and ultrasound to the residual limb.
C. icing and vigorous massage to the residual limb.
D. iontophoresis to the distal residual limb using hyaluronidase.

C163 C7 3
163. A researcher uses a group of volunteers (healthy, college students) to study the
effects of Swiss ball exercises on ankle ROM and balance scores. Twenty
volunteers participated in the 20 minute ball exercise class 3 times a week for 6
weeks. Measurements were taken at the beginning and end of the sessions.

474800226.doc
50 Practice Questions

Significant differences were found in both sets of scores and reported at the local
PT meeting. Based on this research design, you conclude:
A. Swiss ball exercises are an effective intervention to improve ankle stability
following chronic ankle sprain.
B. the Hawthorne effect may have influenced the outcomes of the study.
C. the reliability of the study was threatened with the introduction of systematic
error of measurement.
D. the validity of the study was threatened with the introduction of sampling bias.

C164 C3 3
164. Your patient has a very large right-sided bacterial pneumonia. Her oxygen level is
dangerously low. The body position that would MOST LIKELY improve her
PaO2 is:
A. left sidelying with the head of the bed in the flat position.
B. prone-lying with the head of the bed in the Trendelenburg position.
C. right sidelying with the head of the bed in the flat position.
D. supine-lying with the head of the bed in the Trendelenburg position.

C165 C1 1
165. An examination of a 46 year-old woman reveals drooping of the shoulder,
rotatory winging of the scapula, an inability to shrug the shoulder, and complaints
of aching in the shoulder. Based on these findings, the cause of these symptoms
would MOST LIKELY be due to:
A. a lesion of CN XI.
B. a lesion of the long thoracic nerve.
C. muscle imbalance.
D. strain of the serratus anterior.

C166 C5 3
166. A 66 year-old patient with spastic hemiplegia was referred to you for ambulation
training. The patient is having difficulty with standing-up from a seated position
due to cocontraction of the quadriceps and hamstrings during the knee and hip
extension phase. You wish to use biofeedback beginning with simple knee
extension exercise in the seated position and progressing to sit-to-stand training.
Initially, the biofeedback protocol should consist of:
A. high detection sensitivity with recording electrodes placed closely together.
B. high detection sensitivity with recording electrodes placed far apart.
C. low detection sensitivity with recording electrodes placed closely together.
D. low detection sensitivity with recording electrodes placed far apart.

C167 C2 2
167. Examination of a 57 year-old man with stroke reveals a loss of pain and
temperature sensation on the left side of the face along with loss of pain and

474800226.doc
Practice Questions 51

temperature sensation on the right side of the body. All other sensations are
normal. You suspect a lesion in the:
A. left cerebral cortex or internal capsule.
B. left posterolateral medulla or pons.
C. midbrain.
D. right cerebral cortex or internal capsule.

C168 C2 2
168. A patient is taking the drug Baclofen to control spasticity following spinal cord
injury. This medication can be expected to decrease muscle tone and pain.
Adverse reactions of concern to the physical therapist can include:
A. drowsiness and muscle weakness.
B. headache with visual auras.
C. hypertension and palpitations.
D. urinary retention and discomfort.

C169 C2 2
169. Symptoms of dysdiadochokinesia, dysmetria, and action tremor can be expected
with a lesion located in the:
A. neocerebellum.
B. spinocerebellum.
C. vermis.
D. vestibulocerebellum.

C170 C3 2
170. The BEST gauge of exercise intensity in a healthy individual is:
A. blood pressure.
B. heart rate.
C. rating of perceived exertion.
D. respiratory rate.

C171 C4 3
171. A ten year-old boy with hemophilia fell and injured himself while skateboarding.
He was admitted to your facility and you see him that afternoon. Examination
reveals a hemarthrosis in his left knee. The BEST initial intervention for this
patient is:
A. instruct in use of hot pack and nonweightbearing exercises.
B. instruct the patient in crutch use to assist in early return to ambulation.
C. prescribe a swimming program to maintain ROM and strength while his is
nonweightbearing.
D. splint the limb and instruct in the use of ice, elevation and rest.

C172 C8 2

474800226.doc
52 Practice Questions

172. A 52 year-old male patient recovering from a fractured hip repaired with ORIF
has recently been discharged home. During a home visit, his wife tells you he
woke up yesterday morning and told her he couldn’t remember much. Upon
examination, you find some mild motor loss in his right hand and anomia. You
affirm the presence of short-term memory loss. Your BEST course of action is to:
A. advise the family to document and record any deficits they notice over the
next week, then report back to you.
B. at present ignore these findings; they are expected following surgical
anesthesia.
C. refer him back to his physician for a diagnostic workup; you suspect multi-
infarct dementia.
D. refer him back to his physician for a diagnostic workup; you suspect presenile
dementia, Alzheimer’s type.

C173 C6 3
173. The primary benefit of residual limb wrapping following lower extremity
amputation is to:
A. decrease adherent scar formation.
B. prevent contractures.
C. prevent lymphedema.
D. prevent phantom limb sensation.

C174 C6 3
174. A 61 year-old patient with a transtibial amputation has a short residual limb. The
BEST choice for prosthetic replacement is a:
A. supracondylar suspension.
B. supracondylar/suprapatellar suspension.
C. Syme’s suspension.
D. thigh corset.

C175 C6 1
175. A physical therapist assistant is ambulating a patient using a three-point crutch
gait. The patient is unsteady and fearful of falling. She does not appear to
understand the correct gait sequence. Your BEST strategy is to:
A. discontinue the session and tell the PTA to try again tomorrow.
B. instruct the PTA to have the patient sit down and use mental rehearsal of the
correct steps.
C. instruct the PTA to keep practicing until the patient achieves success.
D. intervene and teach the correct sequence yourself since the PTA is apparently
unable to deal with this special situation.

C176 C1 1

474800226.doc
Practice Questions 53

176. You have access to the x-rays from a patient you have been treating with a Grade
2 spondylolisthesis. The BEST radiographic view to observe this bony anomaly
is:
A. anteroposterior.
B. lateral.
C. oblique.
D. posteroanterior.

C177 C5 3
177. A 38 year-old athlete presents with pain and muscle spasm of the upper back (C7-
T8) extending to the lateral border of the scapula. This encompasses a 10 x 10 cm
area on both sides of the spine. If your ultrasound unit only has a 5 cm2
soundhead, you should treat:
A. both sides in five minutes.
B. both sides in ten minutes.
C. the area in two sections allotting five minutes for each section.
D. the area in two sections allotting two and a half minutes for each area.

C178 C5 2
178. A patient with chronic cervical pain is referred to your clinic. Past medical history
reveals: appendectomy, 12 years ago; chronic heart disease; demand-type
pacemaker, 8 years ago; whiplash injury, 2 years ago. Presently the patient
complains of pain and muscle spasm in the cervical region. The modality that
should not be considered in the case is:
A. hot pack.
B. mechanical traction.
C. transcutaneous electrical stimulation.
D. ultrasound.

C179 C2 2
179. In posturography, patients who sway more or fall only under conditions with the
eyes closed and platform moving (condition 5) or with the visual surround
moving and platform moving (condition 6) demonstrate:
A. problems with sensory selection.
B. somatosensory dependency.
C. vestibular deficiency.
D. visual dependency.

C180 C2 2
180. A patient presented with severe, frequent seizures originating in the medial
temporal lobes. Following bilateral surgical removal of these areas, he is unable
to remember any new information just prior to the surgery to the present. He

474800226.doc
54 Practice Questions

cannot recall text he read minutes ago and cannot remember people he has met
repeatedly. These outcomes are indicative of:
A. a primary deficit from the loss of the amygdala.
B. loss of integration of the temporal lobe with the basal ganglia and frontal
cortex.
C. loss of procedural memory and integration with frontal cortex.
D. loss of the hippocampus and declarative memory function.

C181 C3 2
181. A patient recovering from a myocardial infarction has an exercise capacity of 6
METs. He is in the initial conditioning phase of an outpatient cardiac
rehabilitation program. Progression of his exercise is BEST done by:
A. decreasing frequency first.
B. increasing duration first.
C. increasing frequency first.
D. increasing intensity first.

C182 C3 2
182. While on a home visit, a 9 month-old infant becomes unresponsive. You tell the
mother to call for emergency medical services. Your IMMEDIATE next step is:
A. begin with chest compressions and alternate with rescue breaths at a ratio of
1:7.
B. check for pulse, if no pulse begin chest compressions at a rate of 100
compressions per minute.
C. check for pulse, if no pulse begin chest compressions at a rate of 80
compressions per minute.
D. tilt the infant’s head back and give 2 full breaths, covering both the mouth and
nose with yours.

C183 C4 2
183. A patient in your exercise class develops muscle weakness and fatigue.
Examination reveals leg cramps and hyporeflexia. He also experiences frequent
episodes of postural hypotension and dizziness. Abnormalities on his ECG
include a flat T wave, prolonged QT interval, and depressed ST segment. You
suspect:
A. hyperkalemia
B. hypocalcemia
C. hypokalemia
D. hyponatremia

C184 C8 2
184. A 64 year-old patient with advanced coronary artery disease is receiving
functional mobility training in a P.T. clinic. While walking, she suffers an episode

474800226.doc
Practice Questions 55

of syncope. Through history taking, you determine the MOST LIKELY cause of
her problem is:
A. heart failure.
B. hyperglycemia.
C. postprandial hypertension.
D. seizures.

C185 C6 3
185. The MOST appropriate position to guard a patient who is descending stairs for
the first time using crutches and non-weightbearing on the right is to:
A. stand behind and slightly to the right side.
B. stand behind and slightly to the right side.
C. stand in front and slightly to the left side.
D. stand in front and slightly to the right side.

474800226.doc
56 Practice Questions

C186 C6 3
186. A patient with paraplegia at the T10 level is being discharged home. The rehab
team is assisting the patient’s wife in modifying the home to be barrier-free. A
new first floor bedroom and bathroom wing are being built. The builder asks you
about the entrance. You recommend:
A. a ramp with a grade of 12% and a level landing at top of at least 50 by 50
inches.
B. a ramp with a slope of 1:12 with a level landing at the top of at least 60 by 60
inches.
C. door width of 36 inches with an outside door that opens out.
D. door widths of at least 40 inches with a handrail 34 inches high.

C187 C7 3
187. You have been assigned the job of instructing a new class of home health aides in
how to perform safe transfers. You plan a one day session that includes laboratory
practice. Effective documentation of this session should include:
A. learner’s progress, attainment of learning objectives, and level of satisfaction.
B. objectives, activities and modifications needed, and outcomes.
C. the activities used and the length of time it took for activities to be mastered.
D. the objectives and activities used.

C188 C3 2
188. A 43 year-old patient is recovering from a mild myocardial infarction and is
referred for supervised exercise training. While working out on a treadmill, he
begins to develop ischemia. You will notice:
A. a PVC.
B. bradycardia.
C. ST segment depression on the ECG.
D. ST segment elevation on the ECG.

C189 C8 3
189. A 16 year-old boy with Duchenne’s muscular dystrophy has been confined to
using a power wheelchair for the past three years and is beginning to develop a 10
degree Cobb angle scoliosis. The BEST strategy to help slow this spinal curvature
at this time would be to:
A. alternate the side of the wheelchair power control.
B. emphasize spinal extension exercises.
C. emphasize spinal rotation exercises.
D. order a new wheelchair with a reclining seat back.

C190 C1 2
190. A 55 year-old male presents with a complaint of severe neck and shoulder pain of
two days duration. He reports that he fell asleep on the couch watching TV, and

474800226.doc
Practice Questions 57

has been stiff and sore since. There is tenderness of the cervical muscles on the
right, with increased pain upon palpation. PROM is most limited in flexion, then
sidebending left, and then rotation left, and active extension. Sidebending right
and rotation right is also painful. Based on these examination findings, the
patient’s diagnosis is:
A. cervical radiculopathy.
B. cervical strain.
C. facet syndrome.
D. herniated disc.

C191 C6 2
191. Your patient is a 36 year-old male mail sorter. He complains of numbness and
tingling in his right hand in the median nerve distribution. When you evaluate his
work tasks, you note that he is required to key in the zip codes of about 58 letters
per minute. An appropriate administrative control to decrease his exposure would
be to:
A. provide him with a height adjustable chair to position his wrists and hands in a
neutral alignment.
B. provide the worker with a resting splint to support his wrist.
C. require the worker to attend a cumulative trauma disorder educational class.
D. use job rotation during the workday.

474800226.doc
58 Practice Questions

C192 C5 3
192. A 31 year-old patient with a traumatic brain injury presents with hemiparesis of
the left upper extremity. Your PT examination reveals slight cutaneous and
proprioceptive impairment, fair (3/5) strength of the shoulder muscles and triceps
and slight spasticity of the biceps. Voluntary control of the patient’s left arm has
not progressed as rapidly as you desire. You decide to use functional electrical
stimulation and place the active electrode on the triceps to facilitate active
extension of the elbow. The timing sequence BEST to apply is:
A. 2-second ramp up, 10-second stimulation, no ramp down.
B. 2-second ramp up, 5-second stimulation, 2-second ramp down.
C. 5-second ramp up, 5-second stimulation, 5-second ramp down.
D. no ramp up, 10-second stimulation, 2-second ramp down.

C193 C2 2
193. Following a traumatic brain injury, a patient presents with significant difficulties
in learning new skills. He is wheelchair dependent and will need to learn how to
transfer (a skill he has never done before). The BEST strategy to enhance his
motor learning is to:
A. provide bandwidth feedback using a random practice schedule.
B. provide consistent feedback using a blocked practice schedule.
C. provide summed feedback after every few trials using a serial practice
schedule.
D. use only guided movement to ensure correct performance.

C194 C2 2
194. A patient recovering from a CVA presents with predominant involvement of the
contralateral lower extremity and lesser involvement of the contralateral upper
extremity. He also demonstrates mild apraxia. These clinical manifestations are
characteristic of:
A. anterior cerebral artery syndrome.
B. basilar artery syndrome.
C. middle cerebral artery syndrome.
D. posterior cerebral artery syndrome.

C195 C3 1
195. A patient presents with significant intermittent claudication with onset after two
minutes of walking. On further examination, you expect to find:
A. bright red appearance of the extremity in both elevated and hanging positions.
B. elevation-induced pallor and dependent redness with the extremity in the
hanging position.
C. grayish-white appearance of the extremity in both elevated and hanging
positions.
D. little or no changes in color with changes in extremity position.

474800226.doc
Practice Questions 59

C196 C4 3
196. Following her cesarean section, a patient tells you that she is anxious to return to
her premorbid level of physical activity (working out at the gym 3 days a week
and running 5 miles every other day). Your BEST advice is to tell her to:
A. begin weightlifting after 4 weeks and running after 3 months.
B. begin weightlifting and running after 4 weeks.
C. begin with pelvic floor and gentle abdominal exercises for the first 4-6 weeks.
D. refrain from all exercise and running for at least 6 months.

C197 C8 2
197. A 67 year-old patient lives alone and is referred for home physical therapy
services to improve functional mobility. He refuses to get out of his chair. Upon
examination, he appears irritable, with poor concentration and memory. He tells
you he has been sleeping poorly and has no energy or desire to do anything. He
appears anorexic. The MOST LIKELY explanation for his symptoms is:
A. alcohol abuse.
B. depression.
C. pseudodementia.
D. senile dementia.

474800226.doc
60 Practice Questions

C198 C6 3
198. The rehab team is completing a home visit to recommend environmental
modifications for a 72 year-old patient who is scheduled to be discharged next
week. He is wheelchair dependent. The bathroom has not been adapted. Which of
the following recommendations is NOT appropriate?
A. adding a tub seat and nonskid mat.
B. adding horizontal grab bars positioned at 45 inches.
C. raising the toilet seat to 18 inches.
D. taking the door off to widen the entrance to 32 inches.

C199 C6 2
199. During gait, the lateral pelvic tilt on the side of the swing leg, observed during
frontal plane analysis, serves to:
A. control forward and backward rotations of the pelvis.
B. reduce knee flexion at mid stance.
C. reduce peak rise of the pelvis.
D. reduce physiological valgum at the knee.

C200 C1 2
200. A 40 year-old male experiences central thoracic pain while sitting which
increases during the day. There are no complaints in the night or morning. Lifting
and carrying heavy objects increase the pain. Based on the above information the
MOST LIKELY diagnosis would be:
A. herniated nucleus pulposus of the spine.
B. spondylodiscitis.
C. thoracic postural syndrome.
D. traumatic compression fracture.

474800226.doc
Practice Questions 61

474800226.doc
142

1. crackles and cough


2. heart rate, blood pressure, respiratory rate.
3. product liability.
4. 1:11/2
5. pendulum exercises.
6. stretching the iliopsoas and iliotibial (IT) band; strengthening the abdominals.
7. excessive midtarsal or subtalar pronation.
8. excessive midtarsal or subtalar pronation.
9. hamstrings immediately before the quadriceps to produce cocontraction.
10. there is a normal response on the right while the left is exaggerated and
indicates spastic hypertonia.
11. checking resting BP and HR in sitting, then repeating measurements after
standing for 1 minute.
12. purulent.
13. turning and sitting down.
14. anterior and lateral walls are 21Ú2 to 3 inches higher than the posterior and
medial walls to ensure proper positioning on the ischial seat.
15. their degree of anxiety and attention.
16. 95%.
17. excessive weight gain due to decreased ability to exercise.
18. turn the head and bring the hand to mouth on the same side.
19. tell him to discuss his concerns with the physician.
20. serratus anterior, standing, performing wall push-ups.
21. left posterior rotated innominate.
22. the capitation payment method.
23. brief intense TENS.
24. pelvis.
25. chop, reverse chop with right arm leading.
26. 60 - 90% HRmax.
27. take him back to his room yourself and continue to monitor vital signs.
28. skin blushes.
29. profound aphasia and global deterioration of mental functions.
30. active exercises: sitting and standing, marching in place.
31. have him practice locking the brakes first with his left UE, and then his right.
32. PNF contract-relax technique.
33. decreased A-P to lateral chest ratio.
34. discontinue the treatment; discuss your findings and refer the patient back to
his referring physician.
35. strengthening of middle and lower trapezius and stretching of pectoral muscles.
36. CPT and Common Procedural Coding System procedure codes.
37. extension.
38. carpal tunnel syndrome.

474800226.doc 68
108 Practice Questions

39. Guillain-Barré syndrome.


40. slowing of pulse and increased venous pressure is expected.
41. arterial insufficiency.
42. Functional Reach of 7 inches.
43. bridging, holding with Theraband around both thighs.
44. not use supplemental O2.
45. lightweight wheelchair.
46. trunk flexion with right rotation.
47. soleus and gastrocnemius.
48. the name of those involved, witnesses, what occurred, when it occurred, where
it occurred.
49. pulsed US at 1 MHz.
50. cerebral cortex: right parietal lobe.
51. stocking and glove distribution.
52. a 20 AV heart block.
53. begin with an interval walking program, exercising only to the point of pain.
54. clean and debride the wound in a whirlpool followed by a hydrogel dressing
and pressure relief.
55. is a 17 inch seat height.
56. heel strike.
57. descending stairs.
58. higher than the torque actually generated by the contracting hamstrings.
59. central P/A pressure at a 60 degree angle on the spinous process of T6 while
stabilizing T5.
60. benign paroxysmal positional vertigo.
61. increased heart rate and contractility.
62. hematuria and ecchymosis.
63. highlighting steps with pastels, blues and greens.
64. quadriceps femoris.
65. postpone ambulation and report your findings immediately.
66. asking the child and his parent/caretakers to describe the boy’s most serious
physical problems from their perspectives.
67. anteversion
68. trunk extension and abdominal stabilization exercises.
69. do a comprehensive examination, and if you suspect abuse report your findings
to the appropriate authorities.
70. high volt monophasic pulsed current.
71. prone with no pillow.
72. abduction with elbow extension.
73. the sound side.
74. ECG changes from baseline (3 mm horizontal or downsloping, ST-segment
depression).

474800226.doc
Practice Questions
109

75. agility training.


76. facilitate autolytic debridement.
77. avoid direct exposure to blood and body fluids.
78. knee extensors.
79. have the patient sit down, continue monitoring, and notify the physician.
80. practice stair climbing inside the parallel bars using a 3 inch step.
81. postural drainage, percussion, and shaking to the right lower lobe and other
areas with abnormal auscultatory findings.
82. recognize the patient’s need for hope while outlining realistic short term goals
to improve independence.
83. position the child in a proper sitting position using postural supports.
84. inferior glide at 55 degrees of abduction.
85. capsule-ligamentous pattern of TMJ on the left.
86. deny access to the chart unless written permission by his wife is granted.
87. pneumothorax
88. abduction and internal rotation.
89. reach forward to bear weight with the right arm extended against the wall.
90. Broca’s aphasia.
91. engage in a calming activity and observe behaviors closely.
92. dizziness or syncope upon arising from bed.
93. prolonged endurance training has resulted in a low heart rate.
94. muscle wasting and increased risk of fracture.
95. not use a whirlpool additive.
96. viscoelastic shoe insert with forefoot lateral wedge.
97. transfer to the sound side.
98. a rigid frame.
99. sideward protective extension in sitting.
100. Karvonen’s formula (HR reserve).
101. independent in wheelchair mobility.
102. 9-11.
103. lower cervical flexion.
104. tibial, femoral, and pelvic internal rotation.
105. put a single line through the incorrect date, initial it, document the correct date,
and the date the correction was made.
106. pulse duration.
107. Horner’s syndrome.
108. vestibulocerebellum
109. report this finding to the physician as it might be indicative of peripheral
vascular disease.
110. discontinue treatment and notify his physician immediately.
111. irrigate the wound with povidone-iodine while wearing protective eyewear,
garment, and gloves.

474800226.doc
110 Practice Questions

112. will likely be resistant to activity training if unfamiliar activities are used.
113. provide posterior directed resistance to the right ASIS during stance.
114. prone-on-elbow pushups.
115. affective objectives dealing with attending are not being met.
116. have the same therapist reassess the patients after 6 months.
117. heart rate.
118. explain that this is normal and that the stepping was a newborn reflex which
has gone away.
119. 35-49mm.
120. medial forefoot and rearfoot varus posting.
121. pronator teres syndrome.
122. determine a better treatment time, treat the patient, and bill for the 20 minute
session given.
123. large electrodes, widely spaced.
124. supine, trunk in midline with small pillow under the scapula, arm extended on
supporting pillow, and a small towel roll under the knee.
125. somatosensory integrity.
126. congestive heart failure.
127. intensity prescribed using maximal age-related HR.
128. discontinue UBE exercise; provide massage and active assistive range of
motion for the entire extremity.
129. poor light adaptation.
130. sliding board.
131. lift the front casters and ascend in a wheelie position.
132. decreased bone density.
133. a Pancoast tumor.
134. atraumatic shoulder instability.
135. readjusting the harness and continuing with the treatment.
136. reposition the height of the shelf to below shoulder height.
137. D2 flexion.
138. cushion heel with a rearfoot valgus post.
139. 2+ pulses.
140. hepatitis B.
141. rheumatoid arthritis.
142. prosthetic knee set too far anterior to the TKA line.
143. contoured foam seat.
144. onto her walker and one leg, tuck her pelvis by extending the upper trunk, and
swing her other leg through.
145. involve him in goal setting and have him participate in structuring the training
session.
146. the activities of the non-rehab group were not properly monitored and may
account for these results.

474800226.doc
Practice Questions
111

147. lateral lean toward the right.


148. a solid ankle AFO.
149. delay in achieving developmental milestones.
150. transverse processes of T8.
151. thoracic outlet syndrome.
152. a vertebral artery test.
153. not apply mechanical compression and communicate with the referring
physician.
154. a series of letters traced on the hand.
155. light resistance to forward pelvic rotation during swing.
156. XII
157. inferior wall.
158. all values are abnormal.
159. dermatitis
160. surgical repair.
161. circumduction
162. icing and vigorous massage to the residual limb.
163. the validity of the study was threatened with the introduction of sampling bias.
164. left sidelying with the head of the bed in the flat position.
165. a lesion of CN XI.
166. low detection sensitivity with recording electrodes placed closely together.
167. left posterolateral medulla or pons.
168. drowsiness and muscle weakness.
169. neocerebellum
170. heart rate.
171. splint the limb and instruct in the use of ice, elevation and rest.
172. refer him back to his physician for a diagnostic workup; you suspect multi-
infarct dementia.
173. prevent lymphedema.
174. supracondylar/suprapatellar suspension.
175. instruct the PTA to have the patient sit down and use mental rehearsal of the
correct steps.
176. lateral
177. the area in two sections allotting five minutes for each section.
178. transcutaneous electrical stimulation.
179. vestibular deficiency.
180. loss of the hippocampus and declarative memory function.
181. increasing duration first.
182. tilt the infant’s head back and give 2 full breaths, covering both the mouth and
nose with yours.
183. hypokalemia
184. heart failure.

474800226.doc
112 Practice Questions

185. stand in front and slightly to the right side.


186. a ramp with a slope of 1:12 with a level landing at the top of at least 60 by 60
inches.
187. objectives, activities and modifications needed, and outcomes.
188. ST segment depression on the ECG.
189. alternate the side of the wheelchair power control.
190. facet syndrome.
191. use job rotation during the workday.
192. 5-second ramp up, 5-second stimulation, 5-second ramp down.
193. provide consistent feedback using a blocked practice schedule.
194. anterior cerebral artery syndrome.
195. elevation-induced pallor and dependent redness with the extremity in the
hanging position.
196. begin with pelvic floor and gentle abdominal exercises for the first 4-6 weeks.
197. depression
198. adding horizontal grab bars positioned at 45 inches.
199. reduce peak rise of the pelvis.
200. thoracic postural syndrome.

474800226.doc

Potrebbero piacerti anche